1964 AHSME Problems/Problem 5

Revision as of 11:57, 23 July 2019 by Talkinaway (talk | contribs)
(diff) ← Older revision | Latest revision (diff) | Newer revision → (diff)

Problem

If $y$ varies directly as $x$, and if $y=8$ when $x=4$, the value of $y$ when $x=-8$ is:

$\textbf{(A)}\ -16 \qquad \textbf{(B)}\ -4 \qquad \textbf{(C)}\ -2 \qquad \textbf{(D)}\ 4k, k= \pm1, \pm2, \dots \qquad \\ \textbf{(E)}\ 16k, k=\pm1,\pm2,\dots$


Solution

If $y$ varies directly as $x$, then $\frac{x}{y}$ will be a constant for every pair of $(x, y)$. Thus, we have:

$\frac{4}{8} = \frac{-8}{y}$

$y \cdot \frac{1}{2} = -8$

$y = -16$

$\boxed{\textbf{(A)}}$

See Also

1964 AHSC (ProblemsAnswer KeyResources)
Preceded by
Problem 4
Followed by
Problem 6
1 2 3 4 5 6 7 8 9 10 11 12 13 14 15 16 17 18 19 20 21 22 23 24 25 26 27 28 29 30 31 32 33 34 35 36 37 38 39 40
All AHSME Problems and Solutions

The problems on this page are copyrighted by the Mathematical Association of America's American Mathematics Competitions. AMC logo.png